Đến nội dung

Hình ảnh

Topic tổng hợp một số bất đẳng thức trong kì thi MO các nước

* * * * * 16 Bình chọn

  • Please log in to reply
Chủ đề này có 501 trả lời

#241
khanghaxuan

khanghaxuan

    Trung úy

  • Thành viên
  • 969 Bài viết

Bài 101 : Cho $a,b,c>0$ . CMR : 

$\sum \frac{a}{b+c}+\frac{3}{2}\sqrt[3]{\frac{\sum ab}{\sum a^{2}}}\geq 3$

Spoiler

Mở rộng : 

Tìm hằng số $k$ tốt nhất để bđt sau đúng với mọi $a,b,c>0$ : 

 

$\frac{a}{b+c}+\frac{b}{a+c}+\frac{c}{a+b}+\sqrt[k]{\frac{ab+bc+ca}{a^{2}+b^{2}+c^{2}}}\geq \frac{5}{2}$


Bài viết đã được chỉnh sửa nội dung bởi khanghaxuan: 12-06-2015 - 16:02

Điều tôi muốn biết trước tiên không phải là bạn đã thất bại ra sao mà là bạn đã chấp nhận nó như thế nào .

- A.Lincoln -

#242
khanghaxuan

khanghaxuan

    Trung úy

  • Thành viên
  • 969 Bài viết

Đây là file tổng hợp part 2 :)) 

 

File gửi kèm


Bài viết đã được chỉnh sửa nội dung bởi khanghaxuan: 10-06-2015 - 16:39

Điều tôi muốn biết trước tiên không phải là bạn đã thất bại ra sao mà là bạn đã chấp nhận nó như thế nào .

- A.Lincoln -

#243
longatk08

longatk08

    Sĩ quan

  • Thành viên
  • 350 Bài viết

Bài 101 : Tìm hằng số $k$ tốt nhất để bđt sau đúng với mọi $a,b,c>0$ : 

$\frac{a}{b+c}+\frac{b}{a+c}+\frac{c}{a+b}+\sqrt[k]{\frac{ab+bc+ca}{a^{2}+b^{2}+c^{2}}}\geq \frac{5}{2}$

Những bài thế này hơi mệt đấy nên chọn mấy bài PT thôi để các em nó còn muốn coi :P

Spoiler



#244
longatk08

longatk08

    Sĩ quan

  • Thành viên
  • 350 Bài viết

 

    Vậy $k_{max}=\frac{9}{8}$ là giá trị cần tìm

Sửa một chút về điều kiện bài toán là $a,b,c$ không âm sao cho có ít nhất một số dương và $a+b+c=ab+bc+ac$



#245
Hoang Tung 126

Hoang Tung 126

    Thiếu tá

  • Thành viên
  • 2061 Bài viết

Sửa một chút về điều kiện bài toán là $a,b,c$ không âm sao cho có ít nhất một số dương và $a+b+c=ab+bc+ac$

Nếu thế thì nghe vẻ là bài toán khó hơn đó



#246
khanghaxuan

khanghaxuan

    Trung úy

  • Thành viên
  • 969 Bài viết

Bài 101  đã fix để dễ hơn :P 


Điều tôi muốn biết trước tiên không phải là bạn đã thất bại ra sao mà là bạn đã chấp nhận nó như thế nào .

- A.Lincoln -

#247
khanghaxuan

khanghaxuan

    Trung úy

  • Thành viên
  • 969 Bài viết

Lời  giải bài 101 :  Cho $a,b,c>0$ . CMR : 

$\sum \frac{a}{b+c}+\frac{3}{2}\sqrt[3]{\frac{\sum ab}{\sum a^{2}}}\geq 3$
 

Ta có : $\sum \frac{a}{b+c}=\sum \frac{a^{2}}{ab+ac}\geq \frac{(\sum a)^{2}}{2(\sum ab)}=1+\frac{\sum a^{2}}{2\sum ab}$

Mặt khác ta đánh giá : $\frac{\sum a^{2}}{2\sum ab}+\frac{1}{2}\sqrt[3]{\frac{\sum ab}{\sum a^{2}}}+\frac{1}{2}\sqrt[3]{\frac{\sum ab}{\sum a^{2}}}+\frac{1}{2}\sqrt[3]{\frac{\sum ab}{\sum a^{2}}}$

$\geq 4\sqrt[4]{\frac{\sum a^{2}}{2\sum ab}.\frac{1}{2^{3}.\frac{\sum ab}{\sum a^{2}}}}=2$

Nên $\sum \frac{a}{b+c}+\frac{3}{2}\sqrt[3]{\frac{\sum ab}{\sum a^{2}}}\geq 2+1=3$ (ĐPCM :)) )

Phần mở rộng dành cho các mem suy nghĩ  :closedeyes:


Bài viết đã được chỉnh sửa nội dung bởi khanghaxuan: 12-06-2015 - 16:07

Điều tôi muốn biết trước tiên không phải là bạn đã thất bại ra sao mà là bạn đã chấp nhận nó như thế nào .

- A.Lincoln -

#248
khanghaxuan

khanghaxuan

    Trung úy

  • Thành viên
  • 969 Bài viết

Bài 102 (ELMlO shortlíst 2012) : Cho $a,b,c>0$ thỏa mãn : $a\leq b\leq c$ và $a+b+c=1$ . Chứng minh rằng : 

$\frac{a+c}{\sqrt{a^{2}+c^{2}}}+\frac{b+c}{\sqrt{b^{2}+c^{2}}}+\frac{a+b}{\sqrt{a^{2}+b^{2}}}\leq \frac{3\sqrt{6}(b+c)^{2}}{\sqrt{(a^{2}+b^{2})(b^{2}+c^{2})(c^{2}+a^{2})}}$


Điều tôi muốn biết trước tiên không phải là bạn đã thất bại ra sao mà là bạn đã chấp nhận nó như thế nào .

- A.Lincoln -

#249
Belphegor Varia

Belphegor Varia

    Thượng sĩ

  • Thành viên
  • 227 Bài viết

Bài 103 (VMO 2004) : Xét các số thực dương $x,y,z$ thỏa mãn điều kiện $(x+y+z)^{3}=32xyz$ .

Hãy tìm $\min$ và $\max$ của biểu thức 
$P=\frac{x^{4}+y^{4}+z^{4}}{(x+y+z)^{4}}$


$ \textbf{NMQ}$

Wait a minute, You have enough time. Also tomorrow will come 

Just take off her or give me a ride 

Give me one day or one hour or just one minute for a short word 

 


#250
khanghaxuan

khanghaxuan

    Trung úy

  • Thành viên
  • 969 Bài viết

Lời giải bài 103 :

MIN :

Spoiler

$P=\frac{\sum x^{4}}{(\sum x)^{4}}\geq \frac{xyz(x+y+z)}{(\sum x)^{4}}=\frac{xyz}{(\sum)^{3}}=\frac{1}{32}$

MAX : 

Chuẩn hóa : $x+y+z=1\Rightarrow xyz=\frac{1}{32} $

Do đó : $P=\sum x^{4}=2(\sum xy)^{2}-4(\sum xy)+\frac{5}{4}$

Mà $\frac{1}{3}\geq \sum xy\geq \sqrt{\frac{3}{32}}$

Tới đây xét hàm là xong 

Còn dấu $=$ thì có : $\sum xy , \sum x , xyz$ rồi thì dùng pt bậc 3 tính ra :))


Điều tôi muốn biết trước tiên không phải là bạn đã thất bại ra sao mà là bạn đã chấp nhận nó như thế nào .

- A.Lincoln -

#251
dogsteven

dogsteven

    Đại úy

  • Thành viên
  • 1567 Bài viết

Em xin đóng góp một bài và rồi off game :))

Bài 104. Cho các số thực dương $a,b,c$. Chứng minh rằng: $\dfrac{a}{b}+\dfrac{b}{c}+\dfrac{c}{a}\geqslant 3\sqrt[4]{\dfrac{a^3+b^3+c^3}{3abc}}$


Bài viết đã được chỉnh sửa nội dung bởi ducvipdh12: 12-06-2015 - 20:06

Quyết tâm off dài dài cày hình, số, tổ, rời rạc.


#252
longatk08

longatk08

    Sĩ quan

  • Thành viên
  • 350 Bài viết

Em xin đóng góp một bài và rồi off game :))

Bài 104. Cho các số thực dương $a,b,c$. Chứng minh rằng: $\dfrac{a}{b}+\dfrac{b}{c}+\dfrac{c}{a}\geqslant 3\sqrt[4]{\dfrac{a^3+b^3+c^3}{3abc}}$

Spoiler

Bổ đề:

 

$(x+y+z)^3\geq \frac{4}{27}(x^2y+y^2z+z^2x+xyz)$

 

Trong đó lấy $x=\frac{a}{b},y=\frac{b}{c},z=\frac{c}{a}$ thì ta có:

 

$(\frac{a}{b}+\frac{b}{c}+\frac{c}{a})^3\geq \frac{27}{4}(\frac{a^3+b^3+c^3}{abc}+1)$

 

Do đó ta chỉ cần chứng minh:

 

$\frac{27}{4}(\frac{a^3+b^3+c^3}{abc}+1)\geq 3^3.(\frac{a^3+b^3+c^3}{3abc})^{\frac{3}{4}}$

 

Cái này đặt ẩn rồi biến đổi tương đương được BĐT đúng. :lol:



#253
dogsteven

dogsteven

    Đại úy

  • Thành viên
  • 1567 Bài viết

Spoiler

Bổ đề:

 

$(x+y+z)^3\geq \frac{4}{27}(x^2y+y^2z+z^2x+xyz)$

 

Trong đó lấy $x=\frac{a}{b},y=\frac{b}{c},z=\frac{c}{a}$ thì ta có:

 

$(\frac{a}{b}+\frac{b}{c}+\frac{c}{a})^3\geq \frac{27}{4}(\frac{a^3+b^3+c^3}{abc}+1)$

 

Do đó ta chỉ cần chứng minh:

 

$\frac{27}{4}(\frac{a^3+b^3+c^3}{abc}+1)\geq 3^3.(\frac{a^3+b^3+c^3}{3abc})^{\frac{3}{4}}$

 

Cái này đặt ẩn rồi biến đổi tương đương được BĐT đúng. :lol:

 

Luôn đúng theo AM-GM 4 số: $3.\dfrac{a^3+b^3+c^3}{3abc}+1$ :D


Quyết tâm off dài dài cày hình, số, tổ, rời rạc.


#254
longatk08

longatk08

    Sĩ quan

  • Thành viên
  • 350 Bài viết

Luôn đúng theo AM-GM 4 số: $3.\dfrac{a^3+b^3+c^3}{3abc}+1$ :D

Ơ mình bị lừa...Ôi ngại quá  :wub:



#255
Bui Ba Anh

Bui Ba Anh

    Thiếu úy

  • Thành viên
  • 562 Bài viết

Đây là file tổng hợp part 2 :)) 

Hình như bài số 98 của Ru-ma-ni sai thì phải, nhờ mod xem lại


NgọaLong

#256
hoctrocuaHolmes

hoctrocuaHolmes

    Thượng úy

  • Thành viên
  • 1013 Bài viết

Bài 105: (Japanese 1997) Cho $a,b,c$ là những số thực dương.Chứng minh rằng $\frac{(b+c-a)^{2}}{a^{2}+(b+c)^{2}}+\frac{(a+c-b)^{2}}{b^{2}+(a+c)^{2}}+\frac{(b+a-c)^{2}}{c^{2}+(b+a)^{2}}\geq \frac{3}{5}$



#257
Hoang Nhat Tuan

Hoang Nhat Tuan

    Hỏa Long

  • Thành viên
  • 974 Bài viết

Bài 105: (Japanese 1997) Cho $a,b,c$ là những số thực dương.Chứng minh rằng $\frac{(b+c-a)^{2}}{a^{2}+(b+c)^{2}}+\frac{(a+c-b)^{2}}{b^{2}+(a+c)^{2}}+\frac{(b+a-c)^{2}}{c^{2}+(b+a)^{2}}\geq \frac{3}{5}$

VÌ BĐT là thuần nhất nên ta chuẩn hóa $a+b+c=1$

Ta có:$\frac{(1-2a)^2}{a^2+(1-a)^2}\geq \frac{23-54a}{25}$ ( bằng cách biến đổi tương đương)

Tương tự: $\frac{(1-2b)^2}{b^2+(1-b)^2}\geq \frac{23-54b}{25}$

$\frac{(1-2c)^2}{c^2+(1-c)^2}\geq \frac{23-54c}{25}$

Cộng ba BĐT trên => ĐPCM


Ngài có thể trói cơ thể tôi, buộc tay tôi, điều khiển hành động của tôi: ngài mạnh nhất, và xã hội cho ngài thêm quyền lực; nhưng với ý chí của tôi, thưa ngài, ngài không thể làm gì được.

#258
dogsteven

dogsteven

    Đại úy

  • Thành viên
  • 1567 Bài viết

Bài 105: (Japanese 1997) Cho $a,b,c$ là những số thực dương.Chứng minh rằng $\frac{(b+c-a)^{2}}{a^{2}+(b+c)^{2}}+\frac{(a+c-b)^{2}}{b^{2}+(a+c)^{2}}+\frac{(b+a-c)^{2}}{c^{2}+(b+a)^{2}}\geq \frac{3}{5}$

Bất đẳng thức trên tương đương với: $\sum \dfrac{a(b+c)}{a^2+(b+c)^2}\leqslant \dfrac{6}{5}$

Áp dụng bất đẳng thức Cauchy-Schwarz: $a^2+(b+c)^2\geqslant a(b+c)+\dfrac{3(b+c)^2}{4}$

Do đó ta có $\sum \dfrac{a(b+c)}{a^2+(b+c)^2}\leqslant \sum \dfrac{4a(b+c)}{4a(b+c)+3(b+c)^2}=3-\sum \dfrac{3(b+c)^2}{4a(b+c)+3(b+c)^2}$

Áp dụng bất đẳng thức Cauchy-Schwarz: $\sum \dfrac{(b+c)^2}{4a(b+c)+3(b+c)^2}\geqslant \dfrac{2(a+b+c)^2}{3(a^2+b^2+c^2)+7(ab+bc+ca)}\geqslant \dfrac{3}{5}$

Do đó $3-\sum \dfrac{3(b+c)^2}{4a(b+c)+3(b+c)^2}\leqslant \dfrac{6}{5}$, đây là điều ta cần chứng minh.


Quyết tâm off dài dài cày hình, số, tổ, rời rạc.


#259
Namthemaster1234

Namthemaster1234

    Thiếu úy

  • Thành viên
  • 550 Bài viết

VÌ BĐT là thuần nhất nên ta chuẩn hóa $a+b+c=1$

Ta có:$\frac{(1-2a)^2}{a^2+(1-a)^2}\geq \frac{23-54a}{25}$ ( bằng cách biến đổi tương đương)

Tương tự: $\frac{(1-2b)^2}{b^2+(1-b)^2}\geq \frac{23-54b}{25}$

$\frac{(1-2c)^2}{c^2+(1-c)^2}\geq \frac{23-54c}{25}$

Cộng ba BĐT trên => ĐPCM

 

Nhờ Anh chia sẻ cách tìm bđt tô đỏ trên ạ :luoi:


Đừng lo lắng về khó khăn của bạn trong toán học, tôi đảm bảo với bạn rằng những khó khăn toán học của tôi còn gấp bội.
(Albert Einstein)

Visit my facebook: https://www.facebook.com/cao.simon.56

:icon6: :icon6: :icon6: :icon6: :icon6:


#260
hoanglong2k

hoanglong2k

    Trung úy

  • Điều hành viên THCS
  • 965 Bài viết

Nhờ Anh chia sẻ cách tìm bđt tô đỏ trên ạ :luoi:

   Phương trình tiếp tuyến tại $x_o$ của $f(x)$ là $y=f'(x_o).(x-x_o)+y_o$ với $y_o$ là giá trị của hàm số tại $x_o$

   Muốn tính $f'(x_o)$ thì em có thể dùng đạo hàm hoặc xài máy tính Casio :v

   Nếu là MS thì $f'(x_o)=d/dx(f(x),x_o)$

   Nếu là ES hoặc VN thì $f'(x_o)=\frac{d}{dx}(f(x))|x=x_o$

   ( Mấy nút đó dưới nút ALPHA á )

   Tính $x_o$ thì công việc còn lại là xài $y=f'(x_o).(x-x_o)+y_o$ mà tính ra thôi :D


Bài viết đã được chỉnh sửa nội dung bởi hoanglong2k: 14-06-2015 - 20:34





0 người đang xem chủ đề

0 thành viên, 0 khách, 0 thành viên ẩn danh